Why can a particle decay into two photons but not one?












11












$begingroup$


I recently read an old physics news about the Higgs boson where it was observed to decay into 2 photons and I was wondering why it wouldn't have decayed into a single photon with the combined energy of 2 photons?










share|cite|improve this question











$endgroup$












  • $begingroup$
    Related: physics.stackexchange.com/q/427466/2451 and links therein.
    $endgroup$
    – Qmechanic
    Jan 13 at 0:05


















11












$begingroup$


I recently read an old physics news about the Higgs boson where it was observed to decay into 2 photons and I was wondering why it wouldn't have decayed into a single photon with the combined energy of 2 photons?










share|cite|improve this question











$endgroup$












  • $begingroup$
    Related: physics.stackexchange.com/q/427466/2451 and links therein.
    $endgroup$
    – Qmechanic
    Jan 13 at 0:05
















11












11








11


2



$begingroup$


I recently read an old physics news about the Higgs boson where it was observed to decay into 2 photons and I was wondering why it wouldn't have decayed into a single photon with the combined energy of 2 photons?










share|cite|improve this question











$endgroup$




I recently read an old physics news about the Higgs boson where it was observed to decay into 2 photons and I was wondering why it wouldn't have decayed into a single photon with the combined energy of 2 photons?







particle-physics photons momentum conservation-laws higgs






share|cite|improve this question















share|cite|improve this question













share|cite|improve this question




share|cite|improve this question








edited Jan 13 at 0:36









Ben Crowell

50.4k6155298




50.4k6155298










asked Jan 12 at 3:42









user6760user6760

2,80111940




2,80111940












  • $begingroup$
    Related: physics.stackexchange.com/q/427466/2451 and links therein.
    $endgroup$
    – Qmechanic
    Jan 13 at 0:05




















  • $begingroup$
    Related: physics.stackexchange.com/q/427466/2451 and links therein.
    $endgroup$
    – Qmechanic
    Jan 13 at 0:05


















$begingroup$
Related: physics.stackexchange.com/q/427466/2451 and links therein.
$endgroup$
– Qmechanic
Jan 13 at 0:05






$begingroup$
Related: physics.stackexchange.com/q/427466/2451 and links therein.
$endgroup$
– Qmechanic
Jan 13 at 0:05












2 Answers
2






active

oldest

votes


















39












$begingroup$

No massive particle can decay into a single photon.



In its rest frame, a particle with mass $M$ has momentum $p=0$. If it decayed to a single photon, conservation of energy would require the photon energy to be $E=Mc^2$, while conservation of momentum would require the photon to maintain $p=0$. However, photons obey $E=pc$ (which is the special case of $E^2 = (pc)^2 + (mc^2)^2$ for massless particles). It's not possible to satisfy all these constraints at once. Composite particles may emit single photons, but no massive particle may decay to a photon.






share|cite|improve this answer









$endgroup$













  • $begingroup$
    Massive particle as in fermion with half integer spin right, so it have to decay into some other particles on top of a photon to conserve energy and spin momentum is this what you are saying?
    $endgroup$
    – user6760
    Jan 12 at 6:04








  • 3




    $begingroup$
    @user6760 This argument is about linear, not angular, momentum. And there are plenty of massive particles which obey Bose-Einstein statistics and have integer spins.
    $endgroup$
    – rob
    Jan 12 at 7:12





















17












$begingroup$

The Higgs boson has spin $0$. A photon has spin $1$. The total angular momentum cannot change in the decay, so a Higgs boson cannot decay into a single photon, regardless of the energy. But the total angular momentum of two photons can be zero (because their spins can be oriented in opposite directions), so this decay mode can conserve angular momentum.



As emphasized in a comment, conservation of angular momentum is only a necessary condition, not a sufficient one. Please see rob's answer for clarification about this.






share|cite|improve this answer











$endgroup$









  • 1




    $begingroup$
    I just look up spin so spin can be negative
    $endgroup$
    – user6760
    Jan 12 at 4:14











Your Answer





StackExchange.ifUsing("editor", function () {
return StackExchange.using("mathjaxEditing", function () {
StackExchange.MarkdownEditor.creationCallbacks.add(function (editor, postfix) {
StackExchange.mathjaxEditing.prepareWmdForMathJax(editor, postfix, [["$", "$"], ["\\(","\\)"]]);
});
});
}, "mathjax-editing");

StackExchange.ready(function() {
var channelOptions = {
tags: "".split(" "),
id: "151"
};
initTagRenderer("".split(" "), "".split(" "), channelOptions);

StackExchange.using("externalEditor", function() {
// Have to fire editor after snippets, if snippets enabled
if (StackExchange.settings.snippets.snippetsEnabled) {
StackExchange.using("snippets", function() {
createEditor();
});
}
else {
createEditor();
}
});

function createEditor() {
StackExchange.prepareEditor({
heartbeatType: 'answer',
autoActivateHeartbeat: false,
convertImagesToLinks: false,
noModals: true,
showLowRepImageUploadWarning: true,
reputationToPostImages: null,
bindNavPrevention: true,
postfix: "",
imageUploader: {
brandingHtml: "Powered by u003ca class="icon-imgur-white" href="https://imgur.com/"u003eu003c/au003e",
contentPolicyHtml: "User contributions licensed under u003ca href="https://creativecommons.org/licenses/by-sa/3.0/"u003ecc by-sa 3.0 with attribution requiredu003c/au003e u003ca href="https://stackoverflow.com/legal/content-policy"u003e(content policy)u003c/au003e",
allowUrls: true
},
noCode: true, onDemand: true,
discardSelector: ".discard-answer"
,immediatelyShowMarkdownHelp:true
});


}
});














draft saved

draft discarded


















StackExchange.ready(
function () {
StackExchange.openid.initPostLogin('.new-post-login', 'https%3a%2f%2fphysics.stackexchange.com%2fquestions%2f453657%2fwhy-can-a-particle-decay-into-two-photons-but-not-one%23new-answer', 'question_page');
}
);

Post as a guest















Required, but never shown

























2 Answers
2






active

oldest

votes








2 Answers
2






active

oldest

votes









active

oldest

votes






active

oldest

votes









39












$begingroup$

No massive particle can decay into a single photon.



In its rest frame, a particle with mass $M$ has momentum $p=0$. If it decayed to a single photon, conservation of energy would require the photon energy to be $E=Mc^2$, while conservation of momentum would require the photon to maintain $p=0$. However, photons obey $E=pc$ (which is the special case of $E^2 = (pc)^2 + (mc^2)^2$ for massless particles). It's not possible to satisfy all these constraints at once. Composite particles may emit single photons, but no massive particle may decay to a photon.






share|cite|improve this answer









$endgroup$













  • $begingroup$
    Massive particle as in fermion with half integer spin right, so it have to decay into some other particles on top of a photon to conserve energy and spin momentum is this what you are saying?
    $endgroup$
    – user6760
    Jan 12 at 6:04








  • 3




    $begingroup$
    @user6760 This argument is about linear, not angular, momentum. And there are plenty of massive particles which obey Bose-Einstein statistics and have integer spins.
    $endgroup$
    – rob
    Jan 12 at 7:12


















39












$begingroup$

No massive particle can decay into a single photon.



In its rest frame, a particle with mass $M$ has momentum $p=0$. If it decayed to a single photon, conservation of energy would require the photon energy to be $E=Mc^2$, while conservation of momentum would require the photon to maintain $p=0$. However, photons obey $E=pc$ (which is the special case of $E^2 = (pc)^2 + (mc^2)^2$ for massless particles). It's not possible to satisfy all these constraints at once. Composite particles may emit single photons, but no massive particle may decay to a photon.






share|cite|improve this answer









$endgroup$













  • $begingroup$
    Massive particle as in fermion with half integer spin right, so it have to decay into some other particles on top of a photon to conserve energy and spin momentum is this what you are saying?
    $endgroup$
    – user6760
    Jan 12 at 6:04








  • 3




    $begingroup$
    @user6760 This argument is about linear, not angular, momentum. And there are plenty of massive particles which obey Bose-Einstein statistics and have integer spins.
    $endgroup$
    – rob
    Jan 12 at 7:12
















39












39








39





$begingroup$

No massive particle can decay into a single photon.



In its rest frame, a particle with mass $M$ has momentum $p=0$. If it decayed to a single photon, conservation of energy would require the photon energy to be $E=Mc^2$, while conservation of momentum would require the photon to maintain $p=0$. However, photons obey $E=pc$ (which is the special case of $E^2 = (pc)^2 + (mc^2)^2$ for massless particles). It's not possible to satisfy all these constraints at once. Composite particles may emit single photons, but no massive particle may decay to a photon.






share|cite|improve this answer









$endgroup$



No massive particle can decay into a single photon.



In its rest frame, a particle with mass $M$ has momentum $p=0$. If it decayed to a single photon, conservation of energy would require the photon energy to be $E=Mc^2$, while conservation of momentum would require the photon to maintain $p=0$. However, photons obey $E=pc$ (which is the special case of $E^2 = (pc)^2 + (mc^2)^2$ for massless particles). It's not possible to satisfy all these constraints at once. Composite particles may emit single photons, but no massive particle may decay to a photon.







share|cite|improve this answer












share|cite|improve this answer



share|cite|improve this answer










answered Jan 12 at 5:37









robrob

40.7k972166




40.7k972166












  • $begingroup$
    Massive particle as in fermion with half integer spin right, so it have to decay into some other particles on top of a photon to conserve energy and spin momentum is this what you are saying?
    $endgroup$
    – user6760
    Jan 12 at 6:04








  • 3




    $begingroup$
    @user6760 This argument is about linear, not angular, momentum. And there are plenty of massive particles which obey Bose-Einstein statistics and have integer spins.
    $endgroup$
    – rob
    Jan 12 at 7:12




















  • $begingroup$
    Massive particle as in fermion with half integer spin right, so it have to decay into some other particles on top of a photon to conserve energy and spin momentum is this what you are saying?
    $endgroup$
    – user6760
    Jan 12 at 6:04








  • 3




    $begingroup$
    @user6760 This argument is about linear, not angular, momentum. And there are plenty of massive particles which obey Bose-Einstein statistics and have integer spins.
    $endgroup$
    – rob
    Jan 12 at 7:12


















$begingroup$
Massive particle as in fermion with half integer spin right, so it have to decay into some other particles on top of a photon to conserve energy and spin momentum is this what you are saying?
$endgroup$
– user6760
Jan 12 at 6:04






$begingroup$
Massive particle as in fermion with half integer spin right, so it have to decay into some other particles on top of a photon to conserve energy and spin momentum is this what you are saying?
$endgroup$
– user6760
Jan 12 at 6:04






3




3




$begingroup$
@user6760 This argument is about linear, not angular, momentum. And there are plenty of massive particles which obey Bose-Einstein statistics and have integer spins.
$endgroup$
– rob
Jan 12 at 7:12






$begingroup$
@user6760 This argument is about linear, not angular, momentum. And there are plenty of massive particles which obey Bose-Einstein statistics and have integer spins.
$endgroup$
– rob
Jan 12 at 7:12













17












$begingroup$

The Higgs boson has spin $0$. A photon has spin $1$. The total angular momentum cannot change in the decay, so a Higgs boson cannot decay into a single photon, regardless of the energy. But the total angular momentum of two photons can be zero (because their spins can be oriented in opposite directions), so this decay mode can conserve angular momentum.



As emphasized in a comment, conservation of angular momentum is only a necessary condition, not a sufficient one. Please see rob's answer for clarification about this.






share|cite|improve this answer











$endgroup$









  • 1




    $begingroup$
    I just look up spin so spin can be negative
    $endgroup$
    – user6760
    Jan 12 at 4:14
















17












$begingroup$

The Higgs boson has spin $0$. A photon has spin $1$. The total angular momentum cannot change in the decay, so a Higgs boson cannot decay into a single photon, regardless of the energy. But the total angular momentum of two photons can be zero (because their spins can be oriented in opposite directions), so this decay mode can conserve angular momentum.



As emphasized in a comment, conservation of angular momentum is only a necessary condition, not a sufficient one. Please see rob's answer for clarification about this.






share|cite|improve this answer











$endgroup$









  • 1




    $begingroup$
    I just look up spin so spin can be negative
    $endgroup$
    – user6760
    Jan 12 at 4:14














17












17








17





$begingroup$

The Higgs boson has spin $0$. A photon has spin $1$. The total angular momentum cannot change in the decay, so a Higgs boson cannot decay into a single photon, regardless of the energy. But the total angular momentum of two photons can be zero (because their spins can be oriented in opposite directions), so this decay mode can conserve angular momentum.



As emphasized in a comment, conservation of angular momentum is only a necessary condition, not a sufficient one. Please see rob's answer for clarification about this.






share|cite|improve this answer











$endgroup$



The Higgs boson has spin $0$. A photon has spin $1$. The total angular momentum cannot change in the decay, so a Higgs boson cannot decay into a single photon, regardless of the energy. But the total angular momentum of two photons can be zero (because their spins can be oriented in opposite directions), so this decay mode can conserve angular momentum.



As emphasized in a comment, conservation of angular momentum is only a necessary condition, not a sufficient one. Please see rob's answer for clarification about this.







share|cite|improve this answer














share|cite|improve this answer



share|cite|improve this answer








edited Jan 12 at 14:33

























answered Jan 12 at 3:52









Dan YandDan Yand

10.2k21538




10.2k21538








  • 1




    $begingroup$
    I just look up spin so spin can be negative
    $endgroup$
    – user6760
    Jan 12 at 4:14














  • 1




    $begingroup$
    I just look up spin so spin can be negative
    $endgroup$
    – user6760
    Jan 12 at 4:14








1




1




$begingroup$
I just look up spin so spin can be negative
$endgroup$
– user6760
Jan 12 at 4:14




$begingroup$
I just look up spin so spin can be negative
$endgroup$
– user6760
Jan 12 at 4:14


















draft saved

draft discarded




















































Thanks for contributing an answer to Physics Stack Exchange!


  • Please be sure to answer the question. Provide details and share your research!

But avoid



  • Asking for help, clarification, or responding to other answers.

  • Making statements based on opinion; back them up with references or personal experience.


Use MathJax to format equations. MathJax reference.


To learn more, see our tips on writing great answers.




draft saved


draft discarded














StackExchange.ready(
function () {
StackExchange.openid.initPostLogin('.new-post-login', 'https%3a%2f%2fphysics.stackexchange.com%2fquestions%2f453657%2fwhy-can-a-particle-decay-into-two-photons-but-not-one%23new-answer', 'question_page');
}
);

Post as a guest















Required, but never shown





















































Required, but never shown














Required, but never shown












Required, but never shown







Required, but never shown

































Required, but never shown














Required, but never shown












Required, but never shown







Required, but never shown







Popular posts from this blog

Can a sorcerer learn a 5th-level spell early by creating spell slots using the Font of Magic feature?

Does disintegrating a polymorphed enemy still kill it after the 2018 errata?

A Topological Invariant for $pi_3(U(n))$